Top Posters
Since Sunday
A
6
j
6
c
5
m
5
C
5
d
5
s
5
n
4
i
4
d
4
d
4
J
4
A free membership is required to access uploaded content. Login or Register.

American Constitutional Law Briefs.docx

Uploaded: 7 years ago
Contributor: doit_the_clown
Category: English Writing
Type: Other
Rating: N/A
Helpful
Unhelpful
Filename:   American Constitutional Law Briefs.docx (176.04 kB)
Page Count: 182
Credit Cost: 1
Views: 74
Last Download: N/A
Transcript
American Constitutional Law Briefs Marbury v Madison pg briefed Facts Marbury was one of the famous midnight judges whose commission had been signed by the Secretary of State Marshall but had not been delivered before the morning that Jefferson took office Procedural Posture Marbury went directly to the Supreme Court to compel Jefferson's Secretary of State Madison to deliver their commissions Issue Whether the Supreme Court has the power to review the legislative acts of the Congress to determine their constitutionality Holding Yes Majority Reasoning Marshall stated that Marbury had a right to his commission once it was signed Further Section of the Judiciary Act of gave the supreme court the right to issue a writ of mandamus against persons holding office Thus it would appear that Marbury has a remedy However Article III Section of the Constitution states that the supreme court shall have original jurisdiction in cases affecting Ambassadors other public Ministers and Consuls and those in which a State shall be a party and in all other cases shall have appellate jurisdiction Since a writ of mandamus is an original action Section of the Judiciary Act must be contrary to the constitution because it allows the Supreme Court to hear cases of original jurisdiction for persons besides ministers consuls and ambassadors etc If the constitution is the supreme will of the people to limit government then the legislature can not alter it at will Thus the legislature can not be allowed to pass laws repugnant to the constitution Since it is the Supreme Court's role to interpret laws and resolve conflicts between competing laws and the Constitution is the supreme law of the land then the Supreme Court has the power to interpret the Constitution and decide if a law passed conflicts with it Martin v Hunter's Lessee pg briefed Facts Fairfax a British loyalist owned land in Virginia A Virginia state law provided for the seizure of Fairfax's lands prior to Virginia granted the land to Hunter Martin was the inheritor of the land from Fairfax Martin claimed that the Peace Treaty of and the Jay Treaty of protected the land from seizure Procedural Posture The Virginia court in the original case found for Hunter The Supreme Court reversed ordering the Virginia court to enter judgment for Martin under the authority granted by Section of the Judiciary Act which gave the Supreme Court the power to review final decisions of the highest state courts rejecting claims based on federal law The Virginia state court refused to comply with the order claiming that Section was unconstitutional and the Supreme Court had no constitutional right to review the final decisions of the state courts The case is again being reviewed by the Supreme Court Issue Whether Section of the Judiciary Act of is constitutionally valid giving the Supreme Court the right to review the final decisions of state courts rejecting claims based on federal law Holding Yes Argument Without Supreme Court review of state court decisions there will be no other mode by which Congress can extend the judicial power of the United States to cases of federal cognizance which arise in the state courts resulting in non-uniformity of decisions among states Argument The constitution does not provide explicitly for Supreme Court review of state court decisions Since it must have been foreseen by the drafters that conflicts would arise the omission is evidence that the framers felt that such a powerful tribunal would produce evils greater than those of the occasional collisions which it would be designed to remedy Thus once an action is brought in state court the federal court's sole remedy is to shift it to a lower federal court before it gets to the final court of the state or simply to advise the high state court that they have improperly interpreted the constitution The states are dually sovereign with the federal government and not subject to the laws of Congress which limit their sovereignty Majority Reasoning In Article III the Supreme Court is given the judicial power which shall extend to all cases Thus it is the nature of the case and not the court of origin that determines whether the Supreme Court has appellate jurisdiction The Supreme Court appellate power is not limited only to cases that come up through the lower federal courts Also the Constitution was designed to operate upon the states themselves and not just the persons within the states Thus the states themselves are not equal sovereigns with the federal government but rather subject to its law- making capability Furthermore even if the state courts do not abuse the power of the constitution they are likely to rule differently on it from state to state Thus the need for uniformity in decisions requires an ultimate single court of last resort which exercises review over all states Lastly there is substantial historical evidence that the framers intended Supreme Court review of state court decisions as well as several previous cases which do so Ex Parte McCardle pg briefed Facts McCardle was a newspaper publisher in the post-civil war south He was imprisoned for sedition Procedural Posture McCardle brought a habeus corpus act under an Act of Congress of which authorized the federal courts to grant habeus corpus to anyone restrained in violation of the Constitution and gave the Supreme Court appellate jurisdiction over such actions However before it was ruled upon on the merits by the Supreme Court Congress passed another Act expressly revoking the appellate jurisdiction for these types of actions that it had previously granted in Issue Whether Congress can take away the jurisdiction of the Supreme Court as to habeus corpus acts which jurisdiction was granted in the Act Holding Yes Argument The appellate jurisdiction of the Supreme Court is derived from Article III Section of the constitution not from acts of Congress Majority Reasoning It is true that the appellate jurisdiction is granted by the constitution but in the same article it is made expressly subject to such exceptions and under such regulations as Congress shall make Thus Congress has the power to expand and limit the scope of the appellate jurisdiction of the Supreme Court Quite simply Congress was acting clearly within its power in both granting and then repealing the specific jurisdiction to review habeus corpus cases from the Circuit Courts pursuant to the Act of The Act of does not affect the appellate jurisdiction with regard to any other cases United States v Klein pg briefed Facts Klein was pardoned by the president for aiding in the civil war rebellion A statute existed that would allow persons who did not aid in the rebellion to recover land seized from them in the Reconstruction Previous case law had held that a presidential pardon was conclusive proof that a person had not committed the crime Procedural Posture A new statute was enacted by Congress while the Klein case was pending appeal reversing the previous tradition of a pardon being proof of non-participation and in fact making it conclusive proof of actual participation In addition the statute purported to remove federal court jurisdiction for all such claims arising from pardons Issue Whether Congress has the constitutional power to enact a statute which limits the jurisdiction of the federal courts particularly the Supreme Court when by limiting said jurisdiction would dictate the outcome of a particular case Holding No Argument Congress has the power under Article III to limit the appellate jurisdiction of the federal courts because of the specific language with such exceptions as the Congress shall make Argument Congress does not have the power to dictate the outcome of any particular case because such would be contrary to the separation of powers structure of the Constitution Majority Reasoning The statute removing jurisdiction in this instance was unconstitutional because it was only a means to an end to affect the outcome of this particular case Dismissing the appeal would allow Congress to prescribe the judgments of the Supreme Court directly The statute prescribed how the court should decide an issue of fact and it denied effect to a Presidential Pardon thus violating the separation of powers Plaut v Spendthrift Farm Inc pg supp briefed Facts A Securities Act violation was committed After Supreme Court dismissal of the first action for being brought outside of the statute of limitations Congress passed a new section A to the Securities Exchange Act extending the statute of limitations in these cases as well as providing for the reinstatement of causes of action that had been dismissed on statute of limitations grounds during the time of pendency of the first action in this case thus re-opening the case Procedural Posture In the first action the Supreme Court dismissed the case for being brought outside of the statute of limitations which it has prescribed in the Rules Congress' enactment of new section A reinstated the action Issue Whether Congress can reinstate a case that has been previously dismissed on statute of limitations grounds Holding No Argument Congress has the power under Article III to modify the appellate jurisdiction of the Supreme Court which includes statutorily reinstating a class of cases Argument Congress' power does not include reinstatement of previously decided cases under new laws Majority Reasoning Scalia stated that Article III not only gives the federal judiciary the power to rule on cases but to decide them subject only to review by superior courts in the Article III hierarchy When retroactive legislation requires its own application in a case already finally adjudicated it does no more and no less than reverse a determination once made in a particular case ' Such power is clearly contrary to what the framers contemplated in the separation of powers It did not matter that the statute was with reference to a general class of cases and not ostensibly to a particular case if it gave the Congress the power to reinstate a case it necessarily interfered with the outcome of a particular case Michigan v Long pg briefed Facts Long was arrested and alleged that his search and seizure rights had been violated Procedural Posture The Michigan State Supreme Court ruled that the police search did violated the Fourth Amendment and the Michigan Constitution's own search and seizure laws Issue Whether the Supreme Court has jurisdiction to review state court judgments which concern federal issues and which are not clearly based on an adequate and independent state law grounds Holding Yes If the state court decision does not indicate clearly and expressly by means of a plain statement that it is alternatively based on bona fide separate adequate and independent grounds the Supreme Court has appellate jurisdiction to review the state court ruling Majority Reasoning O'Connor stated that the Court must assume that there are no adequate grounds when it is not clear from the opinion that there were and the opinion appears to rest primarily on federal law It was necessary out of respect for the independence of state courts that the presumption of adequate state grounds go against the states so as to promote clarity thus avoiding excessive remands and advisory opinions This promotes uniformity in the states interpretation of federal law Dissent Reasoning Stevens stated that it would be better to give the presumption for adequate state grounds to the state because of historical concerns of judicial restraint The Supreme Court should not be involved unless there is a reason to vindicate the federal rights of a party A presumption against independent state grounds would have the Court expounding their understanding of Constitutional law to the legal community instead of primarily sitting to resolve disputes McCulloch v Maryland pg briefed Facts In Congress chartered the Second Bank of the United States which became active in Maryland In the Maryland legislature passed an Act to tax any bank not chartered by the Legislature of Maryland thus taxing the U S Bank The law provided for private remedies against the bank operators of which one was McCulloch Procedural Posture The trial court entered judgment on the basis of an agreed statement of facts that the U S Bank was not chartered by the Maryland legislature and the Maryland Court of Appeal affirmed An appeal was taken by writ of error to the Supreme Court Issue Whether Congress has the power to incorporate a bank and Whether the state of Maryland may without violating the constitution tax the U S Bank Holding Yes No Argument Although Congress does not have the enumerated power to incorporate a bank such power is implied by the necessary and proper language of Article I Section Argument Congress not only does not have the enumerated power to incorporate a bank but furthermore only has the powers that the states as independent sovereigns give to it This is evidenced by the necessary and proper language which should be construed to be a limit on Congressional power implying only strict necessity Majority Reasoning Marshall first noted that the Congressional power established by the Constitution originates from the people not the states Article II should be read in light of the previous Articles of Confederation which were unworkable because of their strict limitations on express Congressional power The Constitution by nature must be general in order to adapt to unforeseen circumstances Thus there must be some implied powers to allow Congress to exercise the broad range of express powers given as means to ends The language necessary and proper should be construed to mean convenient or useful or essential not as things that are absolutely necessary otherwise the word proper would be superfluous and there would be no need to include the word absolutely in the enumeration of powers to the states The necessary and proper language is included among the power of Congress not the limitations and so should be read as enlarging the scope of Congress' powers All means which are appropriate and plainly adapted to the exercise of enumerated powers are constitutional not just those that are strictly necessary As to whether Maryland could tax the federal bank the power to tax something is the power to destroy it Since the states are necessarily inferior to the federal government the states do not have the power to destroy by taxing the federal government The people did not design to make their federal government dependent on the states Gibbons v Ogden pg briefed Facts Gibbons was a former partner turned competitor of Ogden Ogden had a monopoly to operate steamboats on the New York Harbor from New York City to New Jersey and Gibbons was competing with him Ogdens' monopoly was granted by the New York state legislature Gibbon's ferries were licensed as vessels to be employed in the coasting trade under a federal law of Procedural Posture The trial court granted an injunction against Gibbons to stop operating his ferry Gibbons brought an appeal to the Supreme Court on the grounds that the statute granting a monopoly to Ogden was unconstitutional as being repugnant to the commerce power granted to Congress Issue Whether Congress has the power to regulate the navigation of steamboats on the New York harbor between New York and New Jersey to the exclusion of the state of New York Holding Yes Argument The New York law is unconstitutional because it usurps Congress' power to regulate interstate commerce which includes navigation Argument Congress does not have the power to regulate non-commerce events such as navigation Also Congress does not have the power to regulate commerce that occurs internal to a state only that that occurs between two states Majority Reasoning Marshall stated that the common understanding of the word commerce necessarily included navigation Thus Congress has the right to regulate navigation as if it were expressly mentioned in the Constitution Congress has the power to regulate commerce among the several states Among means intermingled with not just between Thus the commerce power extends internal to the states because commerce transactions which can affect the states generally can originate and terminate within the state border boundaries Although it does not extend to transactions which are completely internal to a state the commerce power would be useless if it could not extend beyond the state boundaries because that is where the transactions occur Lastly the commerce power is limited only by the constitution The Congress has the full and exclusive power to make rules by which interstate commerce is to be governed This power is centralized in one body but it can act wherever needed in the states United States v E C Knight Co pg briefed Facts Knight acquired the stock of several other sugar manufacturing companies to control about of the nation's sugar refining capacity Procedural Posture The government brought a civil action under the Sherman Act which provided for penalties for restraint of trade or commerce among the several states to set aside the acquisition The lower court dismissed the action and the government appealed to the Supreme Court Issue Whether Congress had the power under the commerce clause to regulate the monopolization of the means of manufacturing a good Holding No Argument A monopoly of manufacture restrains the free trade or commerce among the states and thus is contrary to the Sherman Act Congress has the power to regulate the monopolization of manufacture because it restrains free trade among the states Argument A monopolization of manufacture is not possible Even if it were such power to control it would necessarily extend to all use of raw materials and thus is beyond what the Sherman Act contemplates Majority Reasoning If monopolization of manufacture could exist it could only have an indirect effect on interstate commerce There is a difference between manufacture and commerce namely that commerce succeeds manufacture Thus controlling manufacture only indirectly controls commerce Congress does not have the power to control manufacture because that would be too intrusive a power necessarily applying to all production of raw materials that could be manufactured into a higher product and then subject to commercial interstate transactions Allowing the power to be construed this broadly would leave no powers for the states to exercise pursuant to the tenth amendment All local commerce would then be subject to federal control Thus the distinction must be made between activities that have a direct affect on commerce which Congress can control and those which have merely and indirect or incidental affect on commerce which the states are left to control Houston E W Texas Ry Co v United States The Shreveport Rate Case pg briefed Facts The railroad had rail lines both within Texas and between Texas and Louisiana As an incentive to promote Texas suppliers to sell to Texas manufacturers the railroad maintained lower rates for traffic within the state of Texas while charging disproportionately high rates for traffic to Louisiana Procedural Posture The Interstate Commerce Commission ICC set rates for the transportation of goods from Texas to Louisiana and ordered the railroad to end its discriminatory practice of maintaining lower rates for traffic within the state The railroad challenged that order appealing to the Supreme Court Issue Whether Congress through the ICC has the power to set the intra-state railroad cargo rates of a carrier that has both intra-state and inter-state lines if such intra-state rates represent an unjust discrimination against inter-state commerce Holding Yes Whenever the interstate and intrastate transactions of carriers are so related that the government of one involves the control of the other it is Congress and not the State that is entitled to prescribe the final and dominant rule for otherwise Congress would be denied the exercise of its constitutional authority Argument Congress has not power to regulate the intra-state rates Argument Congress has the power to regulate intra-state rates if they affect interstate commerce Majority Reasoning Congressional authority extends to interstate carriers as instruments of interstate commerce This necessarily includes the right to control all of their operations that have a close and substantial affect on interstate commerce The fact that the carrier has intra-state business as well does not diminish Congress' power to regulate the interstate portion by preventing injury to it Otherwise the commerce power would have no bite among carriers with both lines Furthermore Congress had the power to affect the intrastate lines in other areas such as safety because it also had an interstate commerce component Thus Congress has the power to foster and protect interstate commerce and to take all measures necessary and appropriate to that end although intrastate transactions may be thereby controlled Notes The current of commerce notion has also been invoked as a practical consideration to allow Congress to regulate portions of interstate commerce that appear to be solely intrastate In Stafford v Wallace the Supreme Court held that individual purchases by middlemen of meat destined for the cities was a part of the current of commerce The purchase by the middlemen was local to the state that they were in but they were simply a part of a greater flow of meat from the West to the East The many transactions viewed as a whole represented interstate commerce on a major scale If the middlemen were unregulated their actions could become an obstacle to free trade Wickard v Filburn pg briefed Facts Filburn was a farmer who grew wheat both for sale and for his own use Under the Agricultural Adjustment Act of Filburn was fined for producing too much wheat for his own consumption Procedural Posture Filburn sought enjoinder of the fine and sued the Secretary of Agriculture Wickard The lower court granted the injunction on other grounds and Wickard appealed Issue Whether Congress has the power to regulate the production of wheat for consumption by the farmer apart from the sale of such wheat commercially Holding Yes Argument The Congress does not have the power under the commerce clause to regulate the production and consumption of wheat because these activities are local in character and at most have an indirect effect on interstate commerce Argument The statute does not regulate production or consumption of wheat but only marketing and even if it goes beyond marketing it is necessary and proper in this case Majority Reasoning The court discarded the direct-indirect approach of Gibbons v Ogden for a more encompassing approach Whether an activity had a local is only one of the facts upon which a decision should be based The test should be based on whether the activity has a substantial economic effect on interstate commerce The consumption of homegrown wheat causes extreme volatility in the national market because it is so variable Although the effect of one farmer may trivial he is part of a nationwide market where the overall effect is not trivial Since this activity has a substantial economic effect on the interstate wheat market Congress has the power to regulate it United States v Darby pg briefed Facts Darby was a Georgia lumber manufacturer who hired labor at less than the minimum wage prescribed in the Fair Labor Standards Act of He was indicted on violating this Act which sought to regulate the hours and wages of employees by prohibiting the sale of the product in interstate commerce Procedural Posture Darby challenged the indictment and the lower District Court quashed it holding that it was unconstitutional because it sought to regulate local manufacturing activities Issue Whether Congress has the power to prohibit shipments of product that are manufactured by employees who are paid less than a prescribed minimum wage and required to work more than a prescribed maximum number of hours Holding Yes Argument The Congress only has the power to regulate prohibit the shipment of products which are bad in themselves such as toxic or stolen articles This prohibition is motivated by the regulation of local wages the control of which has been reserved to the states as police power and so is trampling on the states' rights Argument In its power to regulate interstate commerce Congress has the implied power to exclude from commerce any articles which it perceives to be injurious to the public health and welfare Majority Reasoning The fact that the state has not regulated this type of activity does not preclude the federal government from doing so its powers are not limited by the inaction of the state The motive and purpose behind a regulation are legislative concerns and as long as the power is not exercised beyond the contemplation of the constitution Congress is free to use the commerce power to implement public policy Hammer v Dagenhart which limited Congress' power to regulate only those objects which were themselves harmful is overruled The test for whether Congress can regulate an activity under the commerce power is whether the activity has a substantial effect on the commerce or the exercise of the Congressional power over it Congress may choose the means necessary to achieve this end even if it necessarily involves the control of intrastate actions Perez v United States pg briefed Facts Perez was a loan shark He was convicted under Title II of the Consumer Credit Protection Act which was a federal law prohibiting extortionate loan activities The Act was passed by Congress pursuant to findings that organized crime is interstate in nature and a substantial part of the income for organized crime is generated by extortionate loan activities thus loan sharking is an instrumentality of interstate commerce even where individual transactions are purely intrastate in nature Procedural Posture Perez challenged his conviction on the grounds that the Act was unconstitutional as being an impermissible exercise of the commerce power by Congress Issue Whether Title II of the Consumer Credit Protection Act as construed and applied to Perez is a permissible exercise by Congress of its powers under the Commerce Clause Holding Yes Argument The Act is unconstitutional because it exceeds the limits of the commerce power contemplated by the framers of the Constitution It infringes on the States' police power of their own intrastate crime activities Loan sharking is a local activity not an interstate activity Argument Since loan sharking is a substantial revenue generator for organized crime and organized crime is a nationwide problem that uses interstate commerce as a conduit to conduct illegal transactions loan sharking affects interstate commerce and is thus able to be regulated by Congress The States are not able to deal with this problem individually the federal government needs to provide tools to deal with the problem on a nation-wide level Majority Reasoning The majority accepted Congress' findings on the relationship between loan sharking and organized crime and the effect of organized crime on interstate commerce They stated that the commerce clause reaches protection of the instrumentalities of interstate commerce which included the policing of organized crime Citing to Darby the court reasoned that it was permissible for Congress to regulate a class of activities without proof that the particular intrastate activity that was thereby controlled had an effect on commerce It was proper to consider the total incidence that the class of activities had on commerce rather than to try to carve out exceptions for individual occurrences of the activity that were not proven to be directly related to commerce Even if individual transactions of loan sharking were completely local in nature as a whole they comprised a threat to interstate commerce because of their relation to the interstate activities of organized crime Dissent Reasoning Conviction for loan sharking under the federal law should require proof that the individual was actually involved in interstate activities Otherwise a purely local problem would be regulated by the federal government contrary to the States' police power Loan sharking is only a national problem in the sense that all crime is a national problem There is no distinguishing factor about loan-sharking that lends itself to being a threat to interstate commerce per se United States v Five Gambling Devices pg briefed Facts This case involved three companion proceedings arising from a statute that prohibited the shipment of gambling machines in interstate commerce The statute required the registration and reporting of all gambling machines sold by all manufacturers and dealers in gambling devices not just those that had some nexus to interstate commerce Procedural Posture The lower courts found the statutory interpretation unconstitutional Issue Whether the statute requiring the registration of all gambling devices by all manufacturers and dealers was a permissible exercise of the commerce power when it was interpreted to apply to purely intrastate transactions Holding No Argument The statute should be applied according to its literal terms without any showing that any individual activity be shown to have an actual effect on interstate commerce To have an effective regulation of those gambling machine related activities that do have a relationship to interstate commerce it is necessary to require reporting of all intrastate transactions Argument The statute is unconstitutional because it regulates purely intrastate transactions that can not be shown to have any nexus to interstate commerce Majority Reasoning No precedent of the Court has upheld the power of Congress to enact legislation which penalizes failure to report information concerning acts that have not been shown to be related to interstate commerce Although the Darby bootstrap theory which allows a class of activities to be regulated even though it is over-inclusive is applicable under its own facts the present case is distinguishable on its facts However the traditional limitations of the federal system do not go against the lower courts findings There is no unmistakable intent of Congress to apply the act to the police powers normally reserved for the states United States v Bass pg briefed Facts A man was convicted of possession of a firearm in violation of a provision of the Omnibus Crime Control and Safe Streets Act of which applied to any felon who receives possesses or transports in interstate commerce or affecting commerce any firearm Procedural Posture There had been no showing that the defendant's firearms were commerce- related but the lower court convicted anyway Issue Whether the Omnibus Crime Control and Safe Streets Act of applied to merely the possession or receiving of firearms without a nexus to interstate commerce demonstrated Holding No Argument The commerce limitations in the law applied only to transports and that possession and receipt were punishable without a showing that there was a nexus with commerce Argument The interpretation by the government is unconstitutional because it reaches into purely intrastate activities which have no relations to interstate commerce Majority Reasoning Since the statute was criminal in nature such a broad reading as the government asserted would be too intrusive to the police powers of the states In the absence of clear direction of Congressional intent to do so the court would not construe the statute so broadly as to not require a showing of nexus with commerce Notes In Scarborough v United States the government came prepared to show that the firearm in question had once moved in interstate commerce but did not provide a strong link that the person convicted was involved in any way in interstate commerce or got the firearms after his felony conviction Nevertheless the Supreme Court found that the showing was sufficient to satisfy the statutorily required nexus between the possession of the firearms by a convicted felon and commerce Heart of Atlanta Motel v United States pg briefed Facts The hotel had rooms and was located within ready access to two interstate highways It advertised in national media and was a center for conventions of out of state guests The hotel refused to rent rooms to African Americans Procedural Posture The hotel brought a declaratory judgment action attacking the constitutionality of Title II of the Civil Rights Act of which prohibited discrimination on the basis of race in places of public accommodation and which grounded its authority primarily in the commerce power The District Court upheld the Act and the Hotel appealed Issue Whether application of Title II of the Civil Rights Act of to a motel which serves interstate customers is within the constitutional power of Congress under the Commerce Clause Holding Yes The determinative test of the exercise of power by the Congress under the Commerce Clause is simply whether the activity sought to be regulated is commerce which concerns more States than one' and has a real and substantial relation to the national interest Argument Congress did not have the power to legislate against moral wrongs under the guise of the Commerce Power Even if they did the operation of a motel is purely local in character and thus does not affect interstate commerce Argument Discrimination by hotels has a significant effect on interstate commerce by deterring African Americans to travel Majority Reasoning There is ample evidence in the Congressional record that discrimination by places of public accommodation impair African-Americans' ability to travel thus affecting interstate commerce Thus the Act passed the test of commerce which concerns more States than one and discrimination had a substantial relation to the national interest The court then listed several examples of factual scenarios where the Congress had legitimately exercised the commerce power to police activities which were both immoral and had an adverse affect on interstate commerce That Congress was legislating against moral wrongs rendered its enactments no less valid Furthermore if it is interstate commerce that feels the pinch it does not matter how local the operation that applies the squeeze Thus the commerce power encompasses the regulation of local activities that have an affect on interstate commerce Katzenbach v McClung pg briefed Facts Ollie's BBQ was a family-owned restaurant in Birmingham that seated customers and was located on a state highway near an interstate highway The restaurant received about of food mostly meat in interstate commerce annually However it refused to serve African Americans inside its dining facility They could only order take-out Procedural Posture The restaurant brought this action a sister action to Heart of Atlanta to challenge the constitutionality of Title II of the Civil Rights Act as it related to restaurants The District Court found that the Act provided no basis for relating the operations of a local restaurant to interstate commerce and thus issued an injunction restraining the Act from being enforced against the restaurant concluding that it would lose substantial business Issue Whether such part of Title II of the Civil Rights Act that prohibits discrimination on the basis of race in restaurants which serve interstate travelers or which serve food a substantial portion of which has moved in interstate commerce is constitutional Holding Yes Argument There is no basis for believing that racial discrimination in local restaurants has any affect on interstate commerce Congress has merely created a conclusive presumption that it does without making formal findings in the record that support such an assertion The government should be required to show the connection to interstate commerce on a case-by-case basis The volume of food served at Ollie's BBQ prohibits such a finding Argument Racial discrimination in restaurants has an affect on interstate commerce because it deters African Americans from traveling thus reducing business overall Majority Reasoning Although there were no formal findings made by Congress the testimony contained ample evidence to support a finding that racial discrimination in restaurants had an adverse affect on interstate commerce For instance by deterring travel by African Americans the whole business climate suffers for lack of customers Also discrimination puts an artificial restriction on the free flow of goods The wide unrest over the discrimination has a depressant effect on local businesses making new investment and expansion unfavorable in such a depressed business climate Following Wickard local activities can be said to have a substantial effect on interstate commerce when viewed in aggregation Racial discrimination is not merely a local problem As an aggregation it is a nationwide problem Thus it exercises a substantial economic effect on interstate commerce The lack of formal findings to that effect were not fatal to the Act because there existed enough evidence to conclude that Congress had a rational basis for finding a chosen regulatory scheme necessary to the protection of commerce The Court needed to do no further examination to second-guess Congress' judgment in the light of such evidence Concurrence Reasoning Douglas was reluctant to base his opinion entirely on the Commerce Clause because he felt that the human rights issue at stake was more consequential than the commerce clause could justify Thus he would also support the reasoning under the equal protection clause of the fourteenth amendment because it seemed a much more appropriate grounds for anti- discrimination protection Notes Five years later in Daniel v Paul Justice Black was the sole dissenter against application of Title II of the Civil Rights Act to the Lake Nixon Club in Arkansas The club had a snack bar that refused to serve African Americans and a substantial portion of the food served at the snack bar had traveled in interstate commerce However Black felt that the Act would be justifiable if based on the Fourteenth Amendment but he did not feel that there was an adequate relationship between this snack bar and interstate commerce He was afraid that this finding would stretch the commerce power to regulate any remote country place of recreation in every nook and cranny of every precinct and county everywhere Garcia v San Antonio Metro Transit Auth pg briefed Facts Garcia was a bus driver who worked overtime hours Under the FLSA the SAMTA was required to pay a certain wage and comply with certain overtime standards However four months after the Supreme Court's ruling in National League of Cities that the FLSA did not apply to state government agencies in areas of traditional government functions SAMTA notified its employees that the decision relieved it of its overtime obligations under the FLSA because a municipal public mass-transit system was traditionally a local government function and therefore immune from FLSA Procedural Posture Garcia sued for his overtime pay under the FLSA The District Court found that a municipal operation of a mass transit system was a traditional government function and thus under National League of Cities is exempt from the FLSA wage and overtime obligations Issue Whether Congress has the power under the Commerce Clause to regulate activities and functions that are traditionally an integral part of state government operations Holding Yes The fundamental limitation that the constitutional scheme imposes on Congress' power under the commerce clause to protect states as states from intrusion by federal regulation is a procedural one to be found in the political process - states' and citizens' participation in federal governmental action Majority Reasoning The test of National League of Cities also the third prong of the test in Hodel that Congress may not interfere with traditional state government functions is unworkable There is no meaningful way to determine what is a traditional or integral part of a state government's function and what is not Such an approach has led to artificial results since its enactment History is not a viable grounds for a determination because this prevents meaningful change when necessary as well as being fairly arbitrary Furthermore it requires the unelected judiciary to review legislative decisions based on which policies it likes and dislikes This argument goes contrary to Marbury If Congress has a particular power it does not matter whether it interferes with the laws of the states To find limits on the commerce power the constitution itself must be examined Since there are no express limits the constitution suggests that the structure of the federal government itself is the process by which it is regulated i e by state representatives to the federal government The states' interests are best protected by their own representation in the federal government Since the FLSA is a lesser burden on the states than many other Acts it is evidence of the political pressures on the federal government to protect states' rights Thus National League of Cities is overruled Dissent Reasoning Powell reasoned that the majority rendered the th amendment reservation of power to the states meaningless The balancing test of National League of Cities was best designed to protect the states while allowing the Congress proper power The majority also failed to explain how the states' role in the electoral process protects them in their capacity as states themselves The fact that Congress does not generally exceed its constitutional limits to reach state activities does not make judicial review any less necessary on those occasions that it does The States' rights are a matter of congressional law not legislative grace Congress has passed increasingly more legislation of this type while at the same time losing ground with its local constituents This poses a danger for future stability of the federal government because it undermines the constitutional balance of power between the federal government and the states Furthermore it is clerks and aids who normally draft legislation not the Senators themselves Thus the drafters are even one more step removed from the constituents who best know how to govern their local agencies Since the FLSA is so economically intrusive it clearly violates the balance established by National League of Cities Dissent Reasoning O'Connor felt that the majority had backed down from the fight for states' rights just when the states needed help from the Supreme Court There is now a risk that Congress will gradually erase the diffusion of power between state and nation on which the Framers relied Such a fear is not unwarranted given the amount of similar legislative activity in the last years The proper test should be weighing the states' rights as a relevant consideration in determining the constitutionality of uses of the commerce power United States v Lopez pg Supp briefed Facts Lopez was a high school senior in San Antonio who was caught with a caliber handgun and five bullets on school grounds Procedural Posture Lopez was charged with violation of q of the Gun-Free School Zones Act of which made it a federal offense for any individual knowingly to possess a firearm at a school zone The District Court convicted him on a bench trial and sentenced him to six months' imprisonment The Court of Appeal for the th district reversed the conviction on the grounds that the law was unconstitutional as being beyond the power of Congress to legislate control over local public schools and the Supreme Court granted cert Issue Whether q of the Gun-Free School Zones Act is unconstitutional as being beyond the power of Congress to legislate control over local public schools Holding Yes Argument q is valid because possession of a firearm in a school zone substantially affects see Shreveport interstate commerce because it results in violent crime which affects the economy in two ways first it results in increased costs to the taxpayers second it deters people from traveling to areas that are perceived to be unsafe Also the presence of guns in school is a substantial threat to the learning environment which results in a less educated population and therefore a less productive economy Argument The Gun-Free School Zones Act is unconstitutional as being beyond the power of Congress to legislate control over local public schools Majority Reasoning q is a criminal statute that has no observable relationship to commerce or any sort of economic enterprise regardless of how broadly those terms are defined Deterring the presence of guns on school grounds is not part of a larger regulatory scheme to control commerce that would otherwise be undermined if not viewed in the aggregate Furthermore it contains no test by which the firearm in question could be rationally linked to interstate commerce Although the lack of formal Congressional findings to this extent are not fatal their absence fails to save the statute from the otherwise plain observation that it is not commerce-related To accept the government's position that it guns at schools do have a substantial effect on commerce because they adversely affect the economy would be to give virtually unlimited power to Congress to regulate any activity that has a social cost Additionally such a position would allow Congress to provide federally mandated school curriculums and regulate each and every aspect of local schools This is too broad a reading of the commerce power Concurrence Reasoning Kennedy O'Connor Such a broad reading of the commerce power would violate the theory of federalism in which the states perform their role as laboratories for the experimentation with various means of local regulation Concurrence Reasoning Thomas In future cases the Court should take the opportunity to fashion jurisprudence that more accurately reflects the intent of the Framers with respect to the commerce power The substantial effect test is far too broad in light of the historical evidence of the Framers' intent Dissent Reasoning Souter The Court was wrong to second-guess the rational basis for Congress' enactment of this Act based on the commerce clause As a matter of judicial restraint the Court should defer judgment to the legislature when it appears that there is a rational basis for the Act Dissent Reasoning Breyer The economic reality is that the quality of education has a substantial effect on interstate commerce because it affects the individual citizen's ability to compete in the global marketplace Gregory v Ashcroft pg Supp briefed Facts Missouri has a state constitutional provision that provides for the mandatory requirement of judges when they reach age The Age Discrimination in Employment Act prohibits an employer from discharging an individual over the age of for reasons of age However the Act expressly excludes from the definition of employee any person elected to public office or any appointee on the policymaking level Judges in Missouri are first appointed by the Governor and then re- elected Procedural Posture Unknown Issue Whether the Age Discrimination in Employment Act may be applied to the mandatory retirement of Missouri judges Holding No Argument Missouri argued that the judges fell into the exception of the Act as being either elected or policymaking persons and therefore exempt Majority Reasoning Because of the delicate balance of federal vs state power and especially in light of the serious intrusion it would be into state power if the federal government were able to regulate the ages of their public officials O'Connor held that Congress would be taken to have encroached on state power in this context only if there was a plain statement to that effect This was necessary to preserve the Framer's dual sovereignty notion of the power of the states being commensurate with the power of the federal government Especially since the Court in Garcia had left protection of states' rights primarily to the political process of elections we must be absolutely certain that Congress intended such an exercise so that the Commerce Power is kept in check Since the ADEA's exclusion of most public officials is ambiguous the Court would not interpret Congressional intent as being plain enough to effect such a broad exercise of power Concurrence Reasoning The majority's plain statement rule is unwise infeasible and unnecessary to the proper resolution of this case It deviates from the standard set forth in Garcia and there is no reason to think that the rationale of Garcia would be inapplicable here However there is no reason to consider this question because as a matter of simple statutory construction Missouri judges are exempted as elected or policymaking officials New York v United States pg Supp briefed Facts In Congress passed the Low-Level Radioactive Waste Policy Amendments Act of which was intended to solve a national problem of the disposal of low-level radioactive waste by providing a procedure for states to either group together into regional compacts each dumping their waste into a single site in one of the compact states or find their own waste disposal area The Act had three provisions monetary incentives which allowed site states to charge increasingly higher surcharges to non-pact states for disposal of their waste part of which surcharges would be refunded to the states by the Secretary of Energy if they complied with a timeline for finding their own disposal sites access incentives which allowed site states to deny access to non-pact states after a few years and a take-title provision which required the delinquent states to take possession and title of the radioactive waste and assume liability for it if they remained delinquent to the end New York decided to dispose of its own waste and did not join a regional pact However the state had problems locating the site within its borders because the local citizens did not want it Procedural Posture The state of New York brought this action to seek a declaratory judgment that the Act was inconsistent with the Tenth Amendment and the Guarantee Clause of Article IV Issue Whether Congress may direct or otherwise compel a State to regulate a particular private field in a particular way Holding No Argument The th amendment forbids Congress from directly regulating the states to compel them to carry out federal regulation in this private field Although they unquestionably have the power under the Commerce Clause to regulate the generators of the waste they do not have the power to compel the states to directly regulate the waste generators in a particular manner The Act commandeers the legislative processes of the states Furthermore the second part of the act which provides for monetary incentives is beyond Congress' spending power Lastly the Act violates the Guarantee Clause because it attempts to undermine the states' own republican form of government Argument The Constitution's prohibition of convressional directives to state governments can be overcome where the federal interest is sufficiently important to justify state submission Also the Constitution does in some circumstances permit federal directives to state governments Lastly the Constitution envisions a role for Congress as an arbiter of state disputes Majority Reasoning The Tenth Amendment is a truism that simply directs the court to examine what are the internal limitations to the powers granted to Congress in Article I So the court must examine the Commerce Power the Spending Power and the Supremacy Clause The basic premise is that under Hodel Congress may not simply commandeer the state governmental processes Nothing in the Constitution implies that Congress has the ability to require states to govern by federal coercion This premise is supported by looking at the Framer's intent when they chose the structure that the Congress would exercise its power directly over individuals rather than over states as intermediaries Although Congress can motivate or encourage states to regulate in a certain way by making federal assistance conditional or by giving them the choice between doing it themselves or having the federal government do it for them by preemption it can not directly compel This enables state governments to be directly responsive and accountable to the local electorate Where the federal government compels regulation the state officials take the brunt while the federal officials remain insulated thus reducing accountability in the political process Construing the Act in a light most favorable to the United States the take title provision is still clearly beyond Congress' power because Congress neither has the power to force states to take title to the waste thereby subsidizing the generators nor does it have the power to compel regulation That there is a very strong federal interest in controlling waste does not allow Congress to go beyond the Constitution Even if New York state itself agreed to the bargain the state is powerless to waive the Constitutional limits on Congressional power because the Constitution is for the protection of individuals The other parts of the Act are Constitutional because neither monetary incentives nor access denials can reasonably be said to deny a State a republican form of government Concurrence Dissent Reasoning White reasoned that the majority had taken the Act out of its historical context and its contractual setting The states including New York got together to reach their own agreement on how the radioactive waste crisis should be handled They did not seek federal pre-emption but rather federal sanction of their pact under Article I Section which states that no state shall without the consent of Congress enter into any agreement or Compact with another State Thus New York should be estopped from asserting the unconstitutionality of a bargain that it had derived substantial benefit from Bailey v Drexel Furniture pg briefed Facts After the Court held that regulation of child labor was unconstitutional if its basis was the Commerce Power Hammer v Dagenhart Congress passed the Child Labor Tax Law of which imposed a federal excise tax of of the annual net profits of any employer who exceeded the age or working hours limitations provided It was almost identical to the Act declared unconstitutional in Hammer but it rested on the taxing power instead Procedural Posture Drexel brought this action for refund in the District Court after paying under the tax and won The IRS appealed Issue Whether Congress may impose a tax on industries as a means of regulating child labor under the pretext of the taxing power Holding No Argument The Act is a regulation of the employment of child labor internal to the states which is an exclusively state function under the th amendment Argument The Act is a mere excise tax levied by the Congress under its broad power of taxation under the Constitution The court has already gone so far un upholding taxing statutes that it is bound by precedent to uphold this one as well Majority Reasoning The Act on its face appears to be a penalty enacted under the pretext of a tax It provides a heavy burden for departure from a detailed and specific course of conduct It is imposed without regard to the severity or proportion of the violation of the child labor provisions It also requires a mens rea in that the violator knowingly depart from the standards Thus it clearly looks like a penalty To allow it merely because it has the magic word tax would be to break down all constitutional limitations on Congress' power to interfere with state activities because then any subject of federal concern could be regulated by the taxing power Even though some taxes have an incidental penalty-like action this one is primarily a penalty This case is the same as Hammer Also the previous authority relied upon by the government is distinguishable because it involved taxes that were not enacted under a pretext United States v Kahriger pg briefed Facts The Revenue Act of contained a occupational tax on persons engaged in the business of accepting wagers professional interstate gamblers It also required these persons to keep a list of the names and addresses of all employees for public inspection at any time by any state county or municipal agency In the discussions in the Congressional record there was evidence that one of the primary purposes was to tax these professional gamblers out of existence Procedural Posture The lower court found this provision beyond the taxing power of the Congress Issue Whether Congress has the power under the taxing power to enact a tax on a particular profession if the tax also has a regulatory effect which appears to infringe on the states' police power under the th amendment Holding Yes Argument The legislative history indicating a congressional motive to suppress wagering indicates that this is a tax passed under a pretext of revenue generation and thus is not a proper exercise of the taxing power The sole purpose of the law is to penalize gamblers The revenue generation pretext is evidenced by the small amount of revenue actually generated Furthermore a the law requires the gamblers to record and present upon demand the names and addresses of their employees which is clearly an attempt to regulate this occupation Majority Reasoning A federal excise tax is not invalid merely because it discourages or deters the activities taxed Nor is it invalid because the revenue generated is negligible If a tax produces revenue and unless there are penalty provisions extraneous to any tax need the courts are without authority to limit the exercise of the taxing power This tax is not a penalty therefore it is valid The registration requirement simply aids in the collection of the tax United States v Butler pg briefed Facts Butler was a processor of cotton In the Congress passed the Agricultural Adjustment Act as one of the New Deal measures intended to raise agricultural prices by limiting farm production In return for limiting their production the government would give the farmers a subsidy that was raised by taxing the processing stage of the agriculture Procedural Posture Butler attacked the tax on the grounds that it was an integral part of an unconstitutional program to control agricultural production Issue Whether the Act was a valid exercise of the power to spend for the general welfare Holding No Argument Congress has the right to tax and spend to provide for the general welfare This phrase should be liberally construed to cover anything conducive to the national welfare The decision as to what is conducive to the national welfare is the function of Congress alone unreviewable by the courts and this Act was for the general welfare Furthermore it is not coercive because it provides for voluntary compliance through payment of benefits Majority Reasoning Looking to the framer's intent the Court accepted Hamilton's view that Congress has a substantive power to tax and to spend limited only by the requirement that it shall be exercised to provide for the general welfare of the United States However the Court did not reach the determination of whether the Act was for the general welfare because it invades the rights reserved to the states The attempt by congress to regulate the production of agriculture is unconstitutional thus any laws passed as a means to this unconstitutional end are enacted under a pretext The taxing power can not be used to interfere with the states' rights so the spending power should not either The Act is coercive because it does not provide the farmers with a practical choice since their non- compliance would result in their financial ruin Furthermore the power of Congress to contract with individuals is limited by whether its federal power reaches the subject matter of the contract In this case the federal power does not reach so far Dissent The court should only be concerned with the power to enact statutes not their wisdom The court here may be overstepping its bounds by trying to pass judgment on the policy of the law not its consitutionality The coercion argument is unconvincing because Congress must be able to have the power to condition funding on proper use of the funding otherwise its broad powers would be defeated Steward Machine Co v Davis pg briefed Facts Title IX of the Social Security Act imposed a payroll tax on employers but granted a credit of up to of the tax for contributions to a state unemployment fund if such fund was certified by the federal government as meeting the requirements of the Act One of the provisions of the Act required the states to immediately pay over the funds to the federal government who would hold them in trust Procedural Posture Steward sought a refund of the taxes they had paid under this Act Issue Whether Congress has the power to encourage the states to accept federally approved unemployment programs by providing tax credits to employers in that state if the programs are adopted Holding yes Argument The aim of the law is to conscript the state legilatures by the use of economic pressure Thus it is unconstitutional as infringing upon the soveriegnty of the states Argument The statute was designed to enable the states to cooperate with the federal government in the solving of a national problem It does not coerce state governments because it is optional Majority Reasoning Cardozo explained that the unemployment problem was nationwide and was not relieved by the action or inaction of the states Thus the action of the federal government was clearly for the general welfare Congress was capable of determining what was in the best interest of the national welfare in this case There was not coercion but merely motivation or temptation To argue that coercion was the same as economic motivation would be to plunge the law into endless diffifulties because the distinction between what was coercion and what was not would be impossible Congress has the power to tax and to condition the tax on the compliance of the states as long as the subject matter of the tax is related to the scope of national policy and power The Child Labor Law case is distinguishable because in that case the pretext was clearly visible In this case the tax credit would be lawful on its own It is not crippled by the fact that it is tied to conditional performance by the states Woods v Miller Co pg briefed Facts The Housing and Rent Act of was passed under the authority of the war power to regulate the rents of houses in post-WWII America As the soldiers came back from the war they were met with a housing shortage due to the reduction in residential construction The reduction was caused by allocation of building materials to military projects Procedural Posture The District Court held that the authority of Congress to regulate rents by virtue of the war power ended with the President's New-Year's Eve proclamation of peace Also Congress did not state that they were acting under the war power when they passed the Housing and Rent Act The government appealed directly to the Supreme Court Issue Whether the Housing and Rent Act is a constitutional exercise of the war power by Congress Holding Yes Majority Reasoning Douglas Citing to both Hamilton and Ruppert the court stated that the war power includes the power to remedy the evils which have arisen from its rise and progress and continues for the duration of that emergency It does not end with the cessation of hostilities The Presidential proclamation recognized that the state of war still existed and the war effort was what contributed most heavily to the present housing shortage Thus Congress had the power even after the cessation of hostilities to regulate a shortage of housing caused primarily by the war The necessary and proper clause requires that the war power be held over to treat the effects of war Although this holding read broadly would authorize the war power to used during peace to regulate long-term effects of war and swallow up the Ninth and Tenth amendments we must assume that Congress will act responsibly and take into account its constitutional limits when exercising the war power Concurrence Reasoning Jackson felt that the result in this case was clear but was worried about the potential abuse of the war power because it tended to bexercised during periods of hasty patriotism The war power cannot last as long as the effects and consequences of war because many are permanent Missouri v Holland pg briefed Facts There were migratory birds in the northern United States that transited between the U S and Canada These birds had a beneficial effect on the ecosystem by controlling the insect population as well as being a food supply However the birds were being over-hunted The U S and Great Britain entered into a treaty to protect the birds by delineating hunting seasons Pursuant to the treaty the Migratory Bird Treaty Act of was passed by Congress to give effect to the U S side of the treaty Procedural Posture The state of Missouri brought this action to prevent an U S game warden from enforcing the Act on the grounds that it violated the th amendment arguing that Congress did not have to power to pass the Act without the treaty and thus should not be able to pass the Act under the treaty because if the Act standing alone is in violation of the th amendment then the treaty is as well Issue Whether Congress may properly pass an Act that regulates hunting seasons for migratory birds if that Act regulates in traditionally state-controlled areas Holding Yes Majority Reasoning Article II expressly delegates the power of Congress to make treaties Furthermore Article IV declares that treaties made under the authority of the United States are the supreme law of the land If the treaty is valid then it is clearly a necessary and proper action to carry out the treaty-making power in this case The treaty-making power derives from the authority of the United States as an organism itself It does not matter that Congress might not have the power to pass the Act not in pursuance of a treaty because Congress does have the power to make treaties and the Act is a necessary and proper means to give effect to the treaty Since the birds are important and they transit back and forth between the countries the United States has the power to make a treaty concerning their protection and the treaty is valid Since the treaty is valid the Act is valid as being necessary and proper to give effect to a valid treaty Notes In the 's fear that any Constitutional limitation on Congress' power could be overriden by the broad effect given by Holmes to the treaty power in Holland led to a proposed constitutional amendment called the Bricker Amendment which stated that A provision of a treaty which conflicts with this Constitution shall not be of any force or effect and A treaty shall become effective as internal law in the United States only through legislation which would be valid in the absence of a treaty However in Reid v Covert these fears were put to rest by Justice Black when he stated that no agreement can confer power on the Congress which is free from the restraints of the Constitution and that Holland should be read as standing for the proposition that the th amendment is no barrier to the United States' power to make treaties because the states had delegated their rights as to treaties to the federal government Although there is no explicit power in the Constitution for the Congress independent of the treaty power to pass laws concerning foreign affairs it is generally regarded as implied by the fact that the United States' power to interact with other countries must lay in some body and it rests most appropriately in Congress who has the power to make all other federal laws Gibbons v Ogden Part II pg briefed Facts See above Part I Procedural Posture See above Part I Issue Whether a state has the right to pass laws which affect interstate commerce concurrently with that of Congress Holding No Argument The states may severally exercise the same power to regulate commerce within their respective jurisdictions as Congress has with regard to interstate commerce The states possessed this right before the Constitution and so it is reserved to them under the th amendment to the extent that Congress has not acted on it Argument The full power to regulate interstate commerce rests with Congress Thus there is no residual power left to the states because the grant of the whole power is inconsistent with the existence of a residual power The words to regulate require the grant of the full power Majority Reasoning Marshall reasoned that a Congressional power could in some cases be concurrently exercised by the states For example the power to tax was shared by both Congress and the states However the power to regulate interstate commerce could not be shared because it is by its very nature unsharable The states still have the power to pass police laws such as inspection and quarantine laws and the like which act upon a good in preparation for interstate shipment but this is quite different from having the power to pass laws which actually regulate interstate commerce Although the devices by which the power is exercised may appear to be the same between Congress and the states that does not mean that the power is the same Even if a state law encroaches upon a commerce area that the Congress has left untouched the action necessarily interferes with Congress' commerce power if it regulates interstate commerce Thus it is immaterial whether the state law was passed for local police purposes if it conflicts with Congress' ability to exercise the commerce power The law here is then unconstitutional under the Supremacy Clause Wilson v Black Bird Creek Marsh Co pg briefed Facts The company was authorized by Delaware law to put a dam across Black Bird creek which was a navigable waterway flowing into the Delaware river crashed through the dam and was successfully sued by the company for damages Procedural Posture Wilson brought this action to invalidate the Delaware law as being in conflict with Congress' power under the commerce clause to regulate interstate commerce which was conducted on the Creek Issue Whether the Delaware law authorizing the to dam up the navigable waterway was constitutional Holding Yes Majority Reasoning Marshall reasoned that the power to increase the value of the surrounding property as well as the health of the inhabitants was well within the power of the states as long as it did not conflict with the powers of the federal government But since Congress had passed no acts over this creek the repugnancy of the Delaware law must be measured wholly according to its repugnancy to the dormant commerce power In this case the Delaware law can not be considered as repugnant to the dormant commerce power Cooley v Board of Wardens of The Port of Philadelphia pg briefed Facts A Pennsylvania law of required ships entering or leaving Philadelphia harbor to hire a local pilot For failure to comply Cooley was fined The proceeds from the fines went to a fund used to support retired pilots and their dependents Also a congressional statute stated that all previous piloting laws were expressly adopted and the states had the right to enact further similar laws until Congress saw fit to enact laws in this area Procedural Posture Cooley sued for the penalty claiming that the law was unconsitutional as being in conflict with the dormant Commerce power Issue Whether the law was unconsitutional as being in conflict with the dormant Commerce power Holding No Majority Reasoning The regulation of pilots is regulation of navigation and thus regulation of commerce The Act although it expressly adopts existing piloting laws can not grant any more power to the states than does the constitution Thus if the commerce power is exclusive in this area the Act is inoperative and the local law is unconstitutional However since the field of commerce is so diverse it requires laws of varying scope Some facets of interstate commerce require uniform national laws by their very nature Others require purely local legislation to meet diverse needs Those that require uniform national laws must be said to be exclusively regulated by Congress thus barring any state action in that area even when the commerce power is dormant However in this case there is a manifested intent of congress to leave this area of commerce to local regulation Thus this is an example where the commerce power can coexist between the state and federal government if the federal government has not actuall passed a law in that area The determinative factor of whether a state law is repugnant to the constitution in the face of the dormant commerce power is the subject of the regulation not the purpose behind it South Carolina State Hwy Dept v Barnwell Bros pg briefed Facts A South Carolina law prohibited trucks that were more than inches wide or had a gross weight of over pounds from travelling on South Carolina highways About to of the nations trucks exceeded these limits The law was passed to preserve the highways from damage Procedural Posture The trial court found that substantial burdens were put on interstate commerce by this law and that it was an unreasonable means of protecting the highways because it was tied to gross weight instead of axle weight Issue Whether the South Carolina law is unconstitutional as an impermissible conflict with the dormant commerce power Holding No Majority Reasoning There are matters of local state concern the regulation of which unavoidably involves some regulation of interstate commerce but which because of their local character and diversity may not be fully dealt with by congress A state has an adequate local interest in preserving its highways Thus it can pass local laws to regulate the size of shipping on those highways The fact that this necessarily affects interstate commerce is immaterial because so long as the law does not discriminate which this one does not the power is reserved to the states to regulate It is not the judicial function to determine whether the standard is the best approach or not but only to determine whether it is without a rational basis Southern Pacific Co v Arizona pg briefed Facts The Arizona Train Limit Law of prohibited operating railroad trains of more than a prescribed length Reducing the length of the trains was said to increase safety because of less slack action which caused trains to behave uncontrollably However the length limit required the train operators to run about more trains and cost Southern Pacific about a million dollars year in extra costs About of all rail traffic in Arizona was interstate and so it affected train operations from Texas to California Procedural Posture In Arizona sued Sourthern Pacific for the statutory penalties for violating the law The trial court found the law to be an unconstitutional burden on interstate commerce and further found that it was not justified by local safety concerns because the increase in safety by reducing the slack action was offset by the decrease in safety of more trains The state supreme court reversed concluding that a state police law based on safety could not be overturned even though it had a substantial effect on interstate commerce Issue Whether the total effect of the state law as a safety measure in reducing accidents is too small to outweigh the national interest in keeping interstate commerce free of burdens where a uniform national regulation is needed Holding Yes Majority Reasoning The general rule is that the states do not have the authority to substantially impede the free flow of commerce where the need for national uniformity in laws demand that the regulation be done at the national level However this case lies between the two extremes of clearly needing national regulation and clearly needing a local police measure Thus it calls for a balancing of the state and federal interests The findings show that the increase in safety is small if at all Also if the length of trains is to be regulated it must be done uniformly for efficiency Since the Arizona Law is a substantial burden on commerce where a need for uniformity exists and does not have an adequate police justification it is unconstitutional Dissent Reasoning Black thought that the balancing test was best left to the legislature and not the judiciary Douglas felt that the state legislation was adequately tied to safety and thus entitled to a presumption of validity Dean Milk Co v Madison pg briefed Fact Madison Wis has a local ordinance which prohibits the sale within Madison of any milk which has not been pastuerized within a mile radius of the city of Madison The three pastuerizing plants within that radius are subject to rigourous local safety laws Dean Milk operates out of Chicago and its facility meets Federal safety standards Procedural Posture Dean Milk brought this action to strike down the Madison law after it was denied a license to sell milk there The state court rejected the commerce clause attack Issue Whether the discrimination inherent in the Madison ordinance can be justified in view of the character of the local interests and the available methods of protecting them Holding No Argument The regulation of milk in this manner is a substantial burden upon interstate commerce because it plainly discriminates against interstate commerce Argument The ordinance is valid because it is a good-faith attempt to police health of the milk supply It is valid regardless of its affect on interstate commerce because the states have the power to enact local police laws that are not in conflict with existing federal legislation Majority Reasoning The statute clearly is a barrier to interstate commerce It has a discriminatory effect even if it is not purposefully discriminatory If it were held valid simply because it were related to health then the Commerce Clause would be powerless because a state could enact a burdensome and protective law on the pretext of health Since there are alternative methods for assuring the same degree of health protection the local law's interest in health does not outweigh the national interests in non-discriminatory interstate commerce practices It would be just as effective for the local ordinance to require higher standards without requiring local processing A model federal provision existed that would adequately safeguard the public health CTS Corp v Dynamics Corp of Am pg briefed Facts Indiana passed a corporate takeover law which stated that should any party acquire a controlling interest in the number of shares he held he could only acquire voting rights on those shares to the extent approved by a majority vote of the prior disinterested stockholders Procedural Posture The lower court held that the law was unconsitutional as being a hindrance to tender offers and thus an interstate commerce burden Issue Whether the Indiana law is unconstitutional as being in conflict with the dormant Commerce Clause Holding No Argument Tender offers should generally be favored because they represent a shifting of property rights to their highest value use Also the state of Indiana has no interest in protecting non- resident shareholders Majority Reasoning A state has the fundamental right ot pass laws concerning the regulation of corporations it establishes They are only unconsitutional if they discriminate against interstate commerce Since this law has the same effect on interstate commerce as well as intrastate commerce meaning that all shareholders and tender offers are treated the same regardless of locality then it does not discriminate The state regulation of corporations necessarily has some effect on interstate commerce since the shares are traded internationally However there is stability in knowing that the corporation is subject to one set of regulations - that of its home state Concurrence Reasoning Scalia stated that there was no consitutional basis for any balancing test when determining whether a local interest outweighs a federal interest Whether the burden on commerce imposed by a statute is excessive in relation to its benefit is a question for the legislature not the judiciary United Building Constr Trades v Camden pg briefed Facts The city of Camden N J passed a municipal ordinance requiring that at least of the employees of contractors and subcontractors working on city construction projects be Camden residents Procedural Posture challenged the constitutionality of that ordinance under the Privileges and Immunities clause The state supreme court of New Jersey rejected the attack on the grounds that the ordinance discriminates on the basis of municipal residency and not state residency and thus declined to broaden the interpretation of the privileges and immunities clause beyond the literal language in the constitution Issue Whether the Camden ordinance is constitutional under the privileges and immunities clause Holding No Argument The clause does not apply to a municipal ordinance By its own language it applies only to state laws which discriminate based on state residency This ordinance discriminates equally against state residents and non-state residents as long as they are non-Camden residents Even if it does apply to municipal ordinances in this case the law is necessary to counteract grave social and economic ills of which the out-of-city employees are a primary cause Because they live off of Camden without residing in Camden these out-of-city employees promote middle-class flight from Camden resulting in a depleted tax base Majority Reasoning First the privileges and immunities clause does apply to strictly municipal ordinances because what would be unconstitutional if done by the state can be no more readily accomplished by the city acting under the authority of the state Also a person who lives out of state will be just as discriminated against regardless of whether the ordinance also discriminates against other state residents The out-of-state resident has no chance to remedy the law by participating in the local political process Determination of validity must therefore proceed under the two-step process First whether the ordinance burdens a fundamental privilege or immunity protected by the clause and second whether there is a good balancing reason for allowing it anyway Although a Commerce Clause analysis can distinguish between cities acting as regulators conflict with the dormant Commerce Clause and cities acting as participants no conflict the privileges and immunities clause imposes a direct restraint on the local legislature without regard to whether they are acting as a participant Clearly the opportunity to seek employment with private employers whether or not they work for the city is a fundamental right protected by the clause As to whether the law is nontheless justified it can only discriminate against out-of-city residents who are shown to constitute a peculiar source of the evil at which the statute is aimed Here there are no findings as to whether the out-of-city residents are an evil with regard to Camden's law thus the case must be remanded for trial and specific findings to that extent Dissent Reasoning The privileges and immunities clause has long been interpreted as applying only to state laws that discriminate against out-of-state residents The majority cites no historical basis for broadening its scope Furthermore the out-of-state resident's interests are adequately protected by the voters who are not residents of Camden but still residents of New Jersey Notes In Supreme Ct of New Hampshire v Piper the court struck down a state law prohibiting non-state residents from being admitted to the state bar under the privileges and immunities clause The court found that Piper's claim involved a privilege because the practice of law is important to the national economy The court also found that there was no substantial justification for the difference in treatment between state residents and others In Edwards v California the court struck down the anti-Okie law which forbade bringing indigent persons into the state The majority opinion relied entirely on the commerce clause but a concurrence by Douglas stated that the privileges and immunities clause was a better rationale because the right of mobility of persons is more fundamental than that of products Pacific Gas Elec Co v State Energy Comm'n gp briefed Facts A California law imposed a moratorium on the certification of nuclear energy plants until a state agency found that there existed a demonstrated means for disposal of high-level radioactive waste generated by the nuclear power plants There was evidence in the State legislative record that the law was passed for economical reasons such as regulating the price of electricity because an increase in the number of nuclear power plants without an increase in radioactive disposal capacity could result in the shutdown of existing plants and the subsequent instability of electricity prices Procedural Posture P G E brought an action for declaratory judgment against the law claiming that it was preemted by the federal Atomic Energy Act of The District Court granted relief the Court of Appeal reversed and P G E appealed to the Supreme Court Issue Whether the California statutory moratorium on the certification of nuclear power plants is preemted by the Atomic Energy Act of Holding No Argument The state law is preempted because it regulates construction of nuclear plants based on safety reasons Since the AEA's primary function is exclusive federal safety regulation there is no room here for the state to regulate The statute conflicts with actual decisions made by Congress and the NRC The statute frustrates the AEA's goal of the development of nuclear technology Argument Although safety regulation of nuclear plants is forbidden a state may completely prohibit new construction until its safety concerns are satisfied by the federal government Majority Reasoning Historically the federal government has taken efforts to ensure that nuclear power is developed and operated safely while leaving police regulations of the economics of electricity generated by nuclear energy to the states themselves This is evidenced by the AEA's own language when it states that nothing in it is to be construed as affecting the authority of any local government to regulate the generation sale or transmission of electric power produced through the use of nuclear facilities Thus the federal government has explicitly left this power to the states it is not impliedly preempted by the mere existence of the AEA However the federal goverment by the AEA has exclusively retained the right to regulate nuclear safety and so the state has no power to regulate in that specific area Since there is legislative history evidence that the law was passed as primarily an economic and not a safety matter the court accepted California's representation that they were not attempting to regulate safety Even though the Congress and the NRC have recently passed legislation that it is safe and permissible to continue to certify new power plants the state is in no way compelled to do so Thus compliance with both the federal and state statutes are possible here and so the statute is not expressly preempted Lastly the even though the AEA's purpose was to promote the safe development and use of nuclear energy that was not to be accomplished at all costs So the statute is not impliedly preempted as being an obstruction of a Congressional purpose Notes In Rice v Santa Fe Elevator Corp Justice Douglas stated that the test in whether a local law was preempted was the intent of Congress In areas that were traditionally state-regulated any action by Congress was presumed not to preempt unless it was the clear and manifest purpose of Congress There were three ways to divine this purpose If the federal scheme of regulation was so pervasive as to infer that Congress left no room for state regulation Where the federal interest is so dominant that it outweighs the state interests and The state law produces a result inconsistent with the federal objective Note the similarity between this analysis of preemption by actual legislation and the Cooley balancing analysis with regard to the dormant commerce clause In Campbell v Hussey Justice Douglas' plurality opinion struck down a local Georgia statute requiring that tobacco that was grown locally according to federal regulations be marked with a white tag tobacco grown out-of-state in Carolina was to be marked with a blue tag He rejected the argument that the Georgia statute was constitutional because it merely supplemented the federal regulations In this case the question of preemption did not need actual conflict between the federal and state law because one could draw an inference from the structure of the federal regulation scheme that there was no room for state augmentation See point in Rice above Compare this result with the commerce clause rejection under the rationale of local discrimination in the Washington Apple case However in Florida Lime Avocado Growers Inc v Paul the court distinguished Campbell by stating that there was neither an actual nor presumed conflict based on the federal regulatory design Barron v Mayor City Council of Baltimore pg briefed Facts Barron was a wharf owner The city of Baltimore in an effort to construct some streets diverted part of the flow of some streams feeding the Baltimore harbor This caused sandbars to form around Barron's wharf making it too shallow for most ships to do business there Procedural Posture Barron sued the city for taking his property for public use without just compensation under the th Amendment The trial court awarded him damages but the court of appeals reversed Issue Whether the guarantee in the th Amendment that private property shall not be taken for public use without just compensation is applicable to state governments as well as the federal government Holding No Majority Reasoning Marshall felt the answer was easy The historical context of the framing of the constitution implied that the general guarantees in the Bill of Rights only applied to the federal government and not state governments The purpose of the constitution was to ordain and establish a federal government not state governments Thus any limitations on that power should be construed as applying to the federal government since states have their own constitutions The structure of the constitution shows that there was a plain line drawn between the powers and limitations of the federal and state governments and so if the framers meant for these limitations to apply to states they could have made such intent clear The bill of rights itself was a guarantee against the encroachment of federal government That is where the fear resided There was no need for security against local governments and so none was asked for Slaughter-House Cases pg briefed Facts A Louisiana law of created a state corporation for the slaughtering of livestock The corporation was given exclusive power to slaughter livestock and all other private slaughterhouses were required to close Independent butchers could use the corporations facilities for a charge but could not conduct independent operations Procedural Posture The butchers not included in the monopoly claimed that the law deprived them of their right to exercise their trade and challenged it under the th and th amendments The highest state court sustained the law Issue Whether the th and th amendments guarantee federal protection of individual rights of all citizens of the United States against discrimination by their own state governments Holding No Majority Reasoning The states have the proper police power to limit slaughter house operations for the health and safety of their residents The meaning of the th and th amendments must be derived from the historical context of the problems they were designed to remedy namely African slavery The Congress after the end of the Civil War sought to strenghten the freedom of the former slaves by passing these amendments The word servitudes in the th amendment refers to personal servitudes not property rights because of the qualifying word involuntary The purpose of the th amendment was thus to etch freedom for slaves into the constitution so that it later would not be questioned or avoided The th amendment was a further step needed to protect former slaves from the black codes The th amendment must be grouped in with the th and th and it was specifically for black suffrage These three amendments were ratified to counteract the specific evils of discrimination against former slaves They did not create any further guarantees of privileges that did not already exist Specifically they only were meant to guarantee federal privileges not state priviliges whatever they may be The priviliges and immunities clause did not create additional rights it merely required states to apply its laws equally to non-state residents as well as state residents Dissent Reasoning Field stated that the privileges and immunities referred to in the th amendment included the right to pursue lawful employment The clause in article section did for the protection of citizens of one state against discrimination by another state what the th amendment does for the protection of every citizen against discrimination by his own state against him Bradley felt that since the language of the th amendment was general in nature and did not claim to protect only blacks that it was meant to secure fundamental rights of any citizen against discrimination by his state Palko v Connecticut pg briefed Facts Palko was convicted of second-degree murder The state of Connecticut appealed his conviction seeking a higher degree conviction This was made possible by the state's local statute that allowed the state to appeal criminal convictions as well as the defendant The second-degree murder conviction was set aside and he was retried and convicted of first degree murder Procedural Posture Palko brought an action to declare the procedural statute unconstitutional as a violation of his th amendment guarantee against double jeopardy Issue Whether the action of the state in this case amounted to double jeopardy prohibited by the th amendment Holding No Argument The retrial violated the th amendment and whatever is forbidded by the th amendment is also forbidden by the th Moreover whatever would have been forbidden to the federal government in the bill of rights is now forbidden to the states by operation of the th amendment Majority Reasoning There is no such general rule that the th amendment incorporates the bill of rights and applies all of its provisions to the states Certain rights such as that of a grand jury indictment and trial by jury are important but have not been applied to the states through the th amendment because they are not fundamental The rights that are absorbed by the th amendment are those which are indespensible to freedom and liberty such as freedom of thought and speech In this particular case the particular procedure used by the state was not so harsh as to prevent the fair administration of criminal justice The state has a right to prosecute a case against a criminal until it ends in a decision that is free from substantial legal error Adamson v California Pg briefed Facts Adamson was convicted of murder During the trial the state had commented to the jury on his failure to take the stand Procedural Posture Adamson claimed that the conviction violated the th amendment because the state's comment amounted to a violation of the th amendment's self-incrimination privilege in a federal proceeding Issue Whether a state's comment at a state criminal trial on the failure of a defendant to take the stand at trial is a violation of the defendant's th amendment privilege against self-incrimination Holding No Argument The th amendment incorporates the th amendment's privilege against self- incrimination and applies it to the states in the same way that the th amendment applies directly to the federal government Majority Reasoning Although the th amendment's due process clause guarantees a right to a fair trial in a state criminal trial there is no ground under Palko to make the self-incrimination privilege one of the fundamental rights that is incorporated in the th amendment and applied to the states Dissent Reasoning Black felt that the full incorporation of the bill of rights into the th amendment was the original purpose of the th amendment and the intent of the amendment's framers The history demonstrates that both those in favor of and against the amendment thought that it was powerful to forbid the states from depriving a citizen of the protections afforded by the bill of rights The process of Twining to expand or contract the applicability of the bill of rights through the th amendment as needed by natural law was more power than the court was granted by the constitution Also the selective application process of Palko was inconsistent with the historical purpose Concurrence Reasoning Frankfurter argued that the th amendment's due process clause has independent potency apart from the bill of rights It does not represent shorthand for the first amendments However in determining which clauses in the first eight amendments are incorporated and which are not the judicial interpretation of which are fundamental is too subjective The relevant question is whether the ciminal proceedings deprived the accused of the due process of law Duncan v Louisiana pg briefed Facts Duncan was convicted of simple battery which in Louisiana was a misdemeanor punishable by years imprisonment and a fine Procedural Posture Duncan sought trial by jury but the Louisiana constitution grants jury trials only in capital punishment or hard labor cases so the trial judge denied the request Issue Whether the federal constitution guarantees the right to a trial by jury under the th amendment through the th amendment in a state criminal trial where a sentence as long as years may be imposed Holding Yes The th amendment guarantees a right of jury trial in all criminal cases which were they to be tried in a federal court would come within the th amendment's guarantee Argument The th amendment makes the jury trial guarantee of the th amendment applicable to the states in cases where a sentence as long as years may be imposed Argument The constitution imposes no duty on a state to guarantee a trial by jury in a state criminal trial regardless of the severity of the punishment available If the trial by jury is guaranteed in state criminal cases it will cast doubt on the integrity of every trial conducted without a jury Also if due process is deemed to include trial by jury then all past interpretations of the th amendment in the federal courts such as a -man jury would then become applicable to states infringing on their ability to experiment Majority Reasoning The test for whether a bill of rights right is incorporated to the states by the th amendment is whether that right is a fundamental right Although there were prior cases stating in dicta that a right to a trial by jury was not fundamental to a fair trial those cases are rejected as being wrong A right to jury trial is granted to criminal defendants in order to prevent oppression by the Government Although there are other countries that have fair criminal justice systems but use no juries ours is not one of them The supporting framework of our criminal justice system relies upon juries for fairness It is true that there are some criminal cases that may be tried without a jury however this is not one of them Concurrence Reasoning Black expressed that he is glad that selective incorporation has worked since Adamson to incorporate most of the Bill of Rights guarantees He goes on to restate his arguments in support of total incorporation Namely that the privileges and immunities clause of the th amendment serves to totally incorporate the Bill of Rights because what more precious privilege can there be that the privilege to claim the protections of our great Bill of Rights He criticizes Harlan's dissent as being too subjective a definition of due process Dissent Reasoning Harlan stated that the due process clause of the th amendment requires that state procedures be fundamentally fair in all respects but it does not require jury trials in criminal cases The historical evidence demonstrates that the framers of the th amendment did not think that they were incorporating the bill of rights The proper analysis should be a gradual process of judicial inclusion and exclusion to ascertain those immutable principles of free government It is improper for the majority to simply incorporate the jury trial clause jot-for-jot with all of its associated baggage of federal judicial interpretation Each case must be analyzed to see whether it was a fair one Notes In Benton v Maryland the court held that the double jeopardy clause was a fundamental ideal and is applicable to the states Since then as a result of selective incorporation almost all criminal process guarantees are applicable to the states In Wolf v Colorado the court incorporated only the core of the th amendment but not the case law interpreting it in federal courts However later in Mapp v Ohio the court changed its mind and incorporation thereafter meant not only incorporating the core of the bill of rights guarantee but applying every detail of the contours of the guarantee as delineated in judicial interpretations the baggage In Williams v Florida the court held that a man jury was not necessary because the function of the jury was fairness and less than men could still be fair In Apodaca v Oregon the court stated that the verdict did not have to be unanimous for the same reasons Lastly in Burch v Louisiana the court stated that a man non-unanimous jury was unconsitutional thus putting a limit on the relaxations of Williams and Apodaca Calder v Bull pg briefed Facts There was a dispute over a will A probate court decree had refused to approve a will The persons who were the beneficiaries of that will had the judgment set aside and a new hearing was granted at which the will was approved There was a Connecticut law that allowed the probate court to be set aside Procedural Posture The persons who would have inherited the property if the will was void brought an action to declare the law setting aside their initial favorable judgment as violating the ex-post facto clause Issue Whether the Connecticut law was valid Holding Yes Reasoning Chase reasoned that there were fundamental liberty reasons why the law was sound The purposes for which the constitution was written was to give effect to a social compact wherein the government was established to protect the natural and preexisting rights of the citizens The nature of these rights determines the limits of the legislative power to infrnge on these rights The government can not have the power to enact leglislation that violates the natural laws of civilized society that it was established to protect even if such natural right is not explicitly mentioned in the constitution An example is this case the government can not violate the right of an antecedent lawful private contract or the right of private property Dissent Reasoning Iredell stated that the citizens had framed their constitution to define the precise boundaries of the leglislative power Thus if the legislature violates this power its act is certainly void However if the legislature passes a law within its consitutional boundaries the judiciary does not have the power to use subjective determinations of what is contrary to natural law to strike it down Lochner v New York pg briefed Facts Lochner was convicted under a New York law prohibiting bakery employees from working more than hours per day or hours per week Procedural Posture Lochner borught this action to attack the New York bakery labor law Issue Whether the New York law was a constitutional regulation of health and safety of a workplace under state police power Holding No Arguments The state has an interest in the health and safety of both the bakery workers as well as the quality of the bread that they make Thus these laws were passed under a valid exercise of the state's police power Majority Reasoning The statute necessarily employs with the right of contract between the employer and employee Thus the power of the state to police the liberty of the individual to contract which is protected by the th amendment See Allgeyer must be balanced against the state's interest There is a limit on the police power of the states Thus the question is whether this law is a reasonable exercise of the police power or an arbitrary interference with the right of personal liberty to contract as he sees fit There is no reasonable right to interfere with the liberty to contract by determining the hours of a baker This law does not involve safety of the baker who in contrast to a miner is as a class intelligent is not threatened by his power to negotiate hours of employment The state's justification for this law under health and safety is a pretext because the public interest is not sufficiently affected by this act There is no demonstrable causal link between labor hours of a baker and the quality of his product or his own health Dissent Reasoning Harlan felt that the people of New York had decided that the health of an average man is endangered if he works more than hours per week Whether or not this is wise is not a question for the court to inquire The only question for the court is whether the means devised by the state are germane to an end which may be lawfully accomplished and have a real or substantial relation to the protection of health of bakers Common experience tells us that there is a logical relationship There is abundant evidence that the workplace of a baker is hazardous to his health Clearly this is not a plain invasion of rights secured by fundamental law Notes Lochner is criticized as being an overly broad interpreteation of the word liberty in the th amendment At common law liberty meant freedom from physical restraint and it did not include freedom of contract as held in Lochner Also Lochner seemed to read the terms property and due process very broadly to cover contractual rights Pennsylvania Coal Co v Mahon pg briefed Facts The coal company deeded the surface land above a mine to Mahon's predecessors in title The deed expressly reserves the right to remove all of the coal udner the land and puts the risk of loss of the surface property on the grantee However a local statute forbids the mining of coal in such a way as to harm a structure used as a dwelling Procedural Posture Mahon brings an action in equity to enjoin the coal company from mining under his house in such a way as to weaken its support Issue Whether the local statute is a valid exercise of the state's police power or is an unconstitutional taking under the th amendment as incorporated through the th amendment and applied to the states Holding Unconstitutional taking Majority Reasoning The question of whether a regulation is a valid exercise of the police power or an unconstitutional taking depends on the particular facts The property being protected here is private property belonging to a single citizen in which there is no public nuisance if it is destroyed The law is not justified as a protection of personal safety The contract itself provided notice of the risks and the grantee still contracted Since coal rights are worthless if the coal can not be mined preventing their mining is a taking because it is tantamount to destroying it If the police power of the states is allowed to abridge the contract rights of parties it will continue until private property disappears completely In general while property may be regulated to a certain extent if regulation goes too far it will be recognized as a taking The loss should not fall on the coal company who provided for this very risk contractually If the state wants more protection for its citizens it can pay for it Dissent Reasoning A restriction imposed to protect the public health safety or morals from danger is not a taking The restriction here is merely the prohibition of a noxious use Just because a few private citizens are enriched does not make the law non-public If the mining were to set free noxious gas there would be no question that the state could prohibit it for the safety of the citizens without paying the miner Lucas v South Carolina Coastal Council pg briefed Facts Lucas bought some beachfront property in for intending to build single-family residences on it At the time he bought is a coastal zone management statute was in effect which regulated the use of certain critical areas in the beachfront areas but Lucas' property was not a critical area However in the state passed another beachfront management act which completely forbade construction seaward of a baseline marked by the highest points of erosion in the last years Unfortunately Lucas property was seaward of the baseline and so he could not build his residential houses on it Procedural Posture Lucas brought an action for compensation claiming that regardless of whether the legislature had acted legitimately in furtherance of some police power objective he was entitled to compensation The trial court agreed finding that the statute deprived Lucas of any reasonable economic use of the lots rendering them valueless The Supreme Court of Carolina reversed finding that when a regulation respecting the use of property is designed to prevent serious public harm no compensation is owing regardless of the regulations effect on the property's value Issue Whether the beachfront management statute was a taking under the th amendment thereby entitling Lucas to compensation Holding Yes Majority Reasoning Scalia first rejected the contention that since the state had amended the statute to provide for special permits that Lucas was still able to apply for this permit thus making the action un-ripe Even if he won a special permit there is still a temporary taking until he does There are two discrete categories of regulatory action that are compensable without looking at the particular facts - physical invasion of property and denying all economically beneficial or productive use of land Regulations that leave the owner of land without economically beneficial or productive options for its use carry with the the heightened risk that private property is being pressed into some form of public service under the guise of mitigating serious public harm However harm preventing and benefit conferring definitions can be made as support of either side of the controversy It is not critical that the legislature have found the regulation to be harm-preventing The appropriate inquiry is whether the regulation deprives the owner of the land of rights that were part of his legal title i e that were not a nuisance or proscribed under normally property law All total regulatory takings of land must be compensated unless the use would be a common-law nuisance anyway Here the land use was lawful and it can not be said that there was some implied limitation on Lucas' use of the land for residential houses Concurrence Reasoning Kennedy reasoned that land is bought and sold all the time with knowledge that it is subject to the state's power to regulate Where there is a taking alleged from regulations which deprive property of all value the test must be whether the deprivation is contrary to reasonable investment-backed expectations Dissent Reasoning Blackmun reasoned that there was no significant taking here and certainly not a total deprivation of economic value The court has unwisely gone against the precedent that the state has the power to prevent any use of its property that it finds harmful and that the state statute is entitled to a presumption of constitutionality The state made findings tjat this was to prevent harm and the court can not simply disregard them Also the new rule that the court fashions - deprivation of all econaomically feasible use itself cannot be determined objectively Finally the court's exception for nuisance is confusing Dissent Reasoning Stevens The court has unwisely departed from the precedent of Mahon which required a look at the individual facts in each case The question of a taking is one of degree and so requiring the dimunition in value of the land to be total is too rigid and too narrow The generation of a general proposition that total regulatory takings must be compensated as a categorical rule is an unwise approach to takings cases Dolan v City of Tigard pg briefed Facts Dolan owned a hardware store set on an upaved lot downtown in the City of Tigard Adjacent to her property was a stream which flooded often causing damage to the downtown area Dolan wished to get a permit to enlarge her store and pave the lot for parking A city planning commission had developed regulations for managing the heavy traffic and flooding in the downtown area by requiring local business owners to donate a portion of their land adjacent to the stream as an unimproved greenbelt and a paved section along their land as a bicycle pedestrian route if they wished to get improvement permits The local legislature had justified the regulations based on findings that more paving would cause more run-off thus requiring the greenbelt and more development would cause more traffic thus requiring the bicycle pedestrian path Procedural Posture Dolan brought an action against the city claiming that the conditional grant of a portion of her land in return for approval of her building permit was an unconstitutional taking The lower courts found that the city's dedication requirements were reasonably related to the public interest in water and traffic management and so the cost should be borne by Dolan for the management of the increased water flow and traffic that her development would bring Issue Whether the local dedication requirement is sufficiently connected to the purpose for the taking i e water and traffic managment Holding No Majority Reasoning One of the principle purposes of the takings clause is to bar the government from forcing individuals to bear public burdens which in all fairness should be borne by the public as a whole Thus under the doctrine of unconstitutional conditions the city may not require an individual to give up her th amendment right of just compensation in exchange for a government granted benefit where the property sought has little or no relationship to the benefit Although there is a nexus between preventing flooding and limiting development along the sides of the creek it is not a sufficiently close nexus to justify an uncompensated taking There must be a reasonable relationship or a rough proportionality between the flooding and the city's taking of the land The required dedication must be related both in nature and extent to the impact of the proposed development Although there is a need to have an adjacent greenway it is not necessary that the city own the property itself Also the bicycle pedestrian walkway is not sufficiently justified by statistics shown by the city who has the burden of proof here Dissent Reasoning Stevens The burden of proof should not lie with the city A statute should be given the presumption of constitutionality putting the burden on the challenger to show that it is not constitutional Furthermore the taking must be viewed from the entirety of the value of the property A commercial developer views these exactions as a business regulation and a cost of doing business They should not be invalidated unless they are sufficient to deter the owner from proceeding with his planned development Griswold v Connecticut pg briefed Facts Griswold was the executive director of planned parenthood He was convicted under a Connecticut statute that made it a crime to assist our counsel someone for the purpose of preventing conception Procedural Posture The state appellate courts affirmed Issue Whether the Connecticut law is a constitutional exercise of the state's police power in view of the substantive due process of the th amendment Holding No Majority Reasoning The court distanced itself from Lochner stating that they do not sit as a super-legislature to determine the wisdom and need of laws that touch economic or social conditions However this law operates directly on the intimate relationship between husband and wife Although there are rights that are not specifically mentioned in the Bill of Rights the court has held that they nevertheless are constitutionally protected For instance the right to choice in education Pierce v Society of Sisters Meyer v Nebraska These rights were derived from the st amendment right of free speech which was held to include the freedom of thought and to teach Without those peripheral rights the express rights would be less secure Thus the st amendment has a penumbra shadow where privacy is protected from governmental intrusion Likewise the rd amendment prohibition against quartering of soldiers and the th amendment prohibition of search and seizure and the th amendment self-incrimination clause all have a penumbra of privacy The th amendment guarantees that the bill of rights is not to be construed as exclusive of other rights retained byt he people This present case lies within the zone of privacy created by these guarantees Concurrence Reasoning Goldberg The due process clause of the th amendment does not incorporate all of the Bill of Rights but it does protect liberty which is those personal rights which are fundamental such as marital privacy The th amendment itself although it is not an independent source of rights incorporated by the th amendment lends strong support The entire fabric of the Constitution and the traditions it represents demonstrate that the marital right of privacy is of the same fundamental importance as the rights specifically enumerated Where there is such a fundamental right being infringed the state must show a compelling interest not merely rational relation The law here is an extremely bad means-ends fit because the state interest in preventing extra-marital relationships is not furthered by criminalizing contraception Harlan felt that the proper analysis was whether this statute infringed on the due process clause of the th amendment because it violated basic values implicit in the concept of ordered liberty like Palko The liberty here is so fundamental that it must be subjected to strict scrutiny Dissent Reasoning Black felt that the word privacy was being substituted for liberty thus he was afraid that the specific guarantees of the bill of rights were being too broadened The government has a right to invade privacy unless prohibited by some constitutional provision Broadening these guarantees has the danger of diluting them because the concept of privacy can be easily narrowed or broadened according to judicial subjectiveness The court's analysis here is too much like Lochner in its attempt to find a natural law basis for constitutional protections not found in the bill of rights Notes Although Justice Douglas disavows Lochner as a guide and instead relies on penumbras of the enumerated constitutional rights Lochner's liberty of contract could possible also be found in a penumbra of the contracts clause thus there is not a significant distinction It is also unclear as to what the scope of the Griswold right of privacy is It is probably narrower than a private autonomy of choice In Eisenstadt v Baird the court took a further step in overturning a statute that prohibited the distribution of contraceptives not just the use as was the case in Griswold even by unmarried couples not just married couples as was the case in Griswold thus broadening the scope of the right of privacy to include the right of an individual to be free from governmental regulation of birth choices Griswold does not reveal at what point a liberty becomes so fundamental as to deserve strict scrutiny rather than just rational relation Roe v Wade pg briefed Facts Roe was a single pregant woman representing a class action suit against a Texas abortion law that made it a crime to procure an abortion except by medical advice for the purpose of saving the life of the mother Procedural Posture The district court held the law unconstitutional under the th amendment Issue Whether the Texas anti-abortion law is constitutional Holding No Argument The woman's right to end her pregnancy is absolute based on the considerable pyschological physical and economic impact that it has on her to bear an unwanted child This absolute right bars any state imposition of criminal penalties for that choice Argument The state's determination to recognize and protect prenatal life from and after conception constitutes a compelling state interest which overrides any right of privacy of the mother The fetus is a person within the meaning of the th amendment and is thereby protected directly by the constitution Majority Reasoning Although the constitution does not explicitly mention the right of privacy

Related Downloads
Explore
Post your homework questions and get free online help from our incredible volunteers
  722 People Browsing
Your Opinion
Which is the best fuel for late night cramming?
Votes: 146

Previous poll results: Do you believe in global warming?